13
$\begingroup$

Let g be an element of $GL_n(\mathbb C)$. We know that there are orthogonal groups $O(\beta)=\{X\in GL_n(\mathbb C) \mid X^t\beta X=\beta\}$ for any $\beta$, invertible symmetric matrix. Though these groups are conjugate since all symmetric bilinear forms over $\mathbb C$ are equivalent. In literature one can find a way to determine when a matrix $X$ satisfies $X^tX=I$ thus an element of the orthogonal group $O(I)$.

My question is as follows: Is there a way to determine if an invertible matrix X belongs to $O(\beta)$ for some $\beta$. To me it seems that I have to check for all conjugates. Sure enough there will be an easier way out!

More precisely I want to know the following: Given $X$ an invertible matrix how to determine $\beta$, an invertible symmetric matrix, so that $X^t\beta X=\beta$?

$\endgroup$
3
  • $\begingroup$ Maybe my intuition is wrong but if we think of $f(\beta) = X^t \beta X$ as a function then basically we want to know if a fixed point exists for which $f(\beta) = \beta$. My first thought goes to some iterative method, start by assuming that $\beta_1 = I$ and computer $\beta_2 = f(\beta_1)$ then plug that value and try again if a fixed point exists then this iteration would/could converge. $\endgroup$
    – Pushpendre
    Jul 2, 2015 at 5:18
  • 9
    $\begingroup$ For the first part: Since you need $\beta X\beta^{-1} = (X^{t})^{-1}$, it is certainly necessary that $X$ and $X^{-1}$ have the same characteristic polynomial. $\endgroup$ Jul 2, 2015 at 7:51
  • $\begingroup$ right! we need to look at self-reciprocal polynomials. And some clever condition does the job. $\endgroup$ Jul 2, 2015 at 8:03

4 Answers 4

12
$\begingroup$

There is a complete characterization of matrices that belong to at least one orthogonal group. It reads as follows over any arbitrary field $\mathbb{F}$ with characteristic different from $2$ (with algebraic closure denoted by $\overline{\mathbb{F}}$:

Given a matrix $M \in \mathrm{GL}_n(\mathbb{F})$, there exists an invertible symmetrix matrix $\beta$ such that $M^T \beta M=\beta$ if and only if, for every $\lambda \in \overline{\mathbb{F}} \setminus \{0,1,-1\}$ and every positive integer $k$, one has $\mathrm{rk}(M-\lambda I_n)^k=\mathrm{rk} (M-\lambda^{-1} I_n)^k$ and, for each one of the (possibly absent) eigenvalues $1$ and $-1$ and every positive integer $k$, there is an even number of Jordan cells of size $2k$ in the Jordan reduction of $M$.

Moreoever, if you have access to the Jordan reduction of $M$ and the above conditions are satisfied, then coming up with an explicit solution $\beta$ is not difficult.

This characterization has been known for a very long time. See my preprint http://arxiv.org/abs/1008.4458 for a recent account using elementary methods.

$\endgroup$
9
$\begingroup$

Consider the entries of $X^t \beta X - \beta$ as linear equations in the entries of the symmetric matrix $\beta$, and solve. For $n \times n$ matrices these are $(n+1)n/2$ equations in $(n+1)n/2$ unknowns. If there is a nontrivial solution, compute the rank of $\beta$ (using generic values for the free variables).

EDIT: It probably should be mentioned that this is a similarity invariant, i.e. if $\beta$ works for $X$ then $S^t \beta S$ works for $S^{-1} X S$ where $S$ is any invertible matrix.

In the cases $n=4$ and $n=5$, I tried companion matrices of self-reciprocal polynomials $p(x) = 1 + a_1 x + a_2 x^2 + a_1 x^3 + x^4$ and $p(x) = 1 + a_1 x + a_2 x^2 + a_2 x^3 + a_1 x^4 + x^5$ respectively. It turns out that for $n=4$ there are invertible solutions $\beta$ iff $a_1 \ne \pm (1 + a_2/2)$, and for $n=5$ iff $a_1 \ne -1 - a_2$.

$\endgroup$
7
$\begingroup$

Interesting question !

Let us denote $L_X:\beta\mapsto X^T\beta X$. This is an endomorphism of ${\bf Sym}_n({\mathbb C})$.

Lemma. The characteristic polynomial of $L_X$ is $$\prod_{i\le j}(t-\mu_i\mu_j)$$ where $\mu_1\ldots,\mu_n$ are the eigenvalues of $X$.

Sketch of the proof : the characteristic polynomial has degree $n(n+1)/2$. Suppose that $X$ is diagonalisable, and that the products $\mu_i\mu_j$ are pairwise distinct. Then each $\mu_i\mu_j$ is an eigenvalue, associated with an eigenvector $\ell^T_i\ell_j+\ell^T_j\ell_i$ where $\ell_iX=\mu_i\ell_i$. So these are all the eigenvalues and the formula is correct in this case. Because the characteristic polynomial of $L_X$ is also a polynomial in the entries of $X$, as well as the product indicated above, the equality stands in the closure (topological or Zariski) of the special case, which is ${\bf M}_n(\mathbb C)$.

Now, what are we looking for ? An eigenvector $\beta$, non-singular and associated with the eigenvalue $1$. Thus we need $\mu_i\mu_j=1$ for some pair. But not only for some, because $\ell^T_i\ell_j+\ell^T_j\ell_i$ is singular (unless $n=2$). What we really need is enough multiplicity that we can pick a non-singular eigen-matrix. If $X$ is diagonalizable, this means that the eigenvalues be associated pairwise: $$\mu_1\mu_n=\mu_2\mu_{n-2}=\cdots=1.$$ Notice that if $n$ is even, this yields the necessary condition that $\det X=1$. It was obvious a priori that $(\det X)^2=1$, but actually the case $-1$ is excluded. This ressembles the well-known fact that symplectic matrices have determinant $+1$.

Edit. A more direct and definitive way to arrive to this conclusion : if $\beta$ exists, then $X^{-T}=\beta X\beta^{-1}$, that is $X^T$ is conjugated to $X^{-1}$. This proves that $\lambda\mapsto\lambda^{-1}$ is an involution of the spectrum of $X$. Conversely, suppose that $X^t$ and $X^{-1}$ are conjugated ; can we choose a symmetric conjugation matrix ? The answer is no, as shown by the counterexample $$X=\begin{pmatrix} 1 & 1 \\ 0 & 1 \end{pmatrix}.$$

$\endgroup$
2
  • $\begingroup$ The eigenvalues do not all need to be paired up; it is possible to have $\det X=-1$. For instance, it is trivial to see that the scalar matrix $-I$ is always orthogonal. More generally, an eigenvalue which is $\pm 1$ can be "paired with itself" in when constructing a nonsingular $\beta$. $\endgroup$ Jul 3, 2015 at 17:57
  • $\begingroup$ I wonder if we can make an if and only if statement using eigen values of X for a particular $\beta$. For example it is great to know that $\beta$ has form $l_i^tl_j + l_j^t l_i$. $\endgroup$ Jul 6, 2015 at 11:23
1
$\begingroup$

Not quite an answer: If you replace "symmetric" by hermitian, then, and transpose by conjugate transpose, then, since every hermitian matrix can be written by as $\beta = A^* A,$ then $X^* \beta X = \beta$ implies that a conjugate of $X$ lies in the unitary group, which is necessary and sufficient. That is the same as saying that the eigenvalues of $X$ are of modulus one (and the matrix is diagonalizable).The same works with "symmetric" and "transpose" but over $\mathbb{R}.$

ADDITION for symmetric matrices, there is a canonical form (due to Horn and Sergeichuk: Canonical forms for complex matrix congruence and ∗congruence Roger A. Horn a,∗ , Vladimir V. Sergeichuk b [Linear Algebra and Applications, 2006]. ) See, equations (7) and on in the paper. There are three types of blocks, and clearly a matrix is in the orthogonal group of some complex symmetric matrix if it is conjugate to a direct sum of matrices in the orthogonal groups of the three block types.

$\endgroup$
4
  • 1
    $\begingroup$ No, first you mean "modulus 1" instead of "roots of unity", second you also need the matrix to be diagonalizable over the complex numbers. $\endgroup$
    – YCor
    Jul 2, 2015 at 9:14
  • $\begingroup$ @YCor Yes, correct. That's what happens when you do this too late at night :) $\endgroup$
    – Igor Rivin
    Jul 2, 2015 at 12:56
  • 2
    $\begingroup$ The symmetric case seems to be much more complicated, though. For instance, it is easy to see that a diagonal matrix is in some $O(\beta)$ iff its eigenvalues other than $\pm1$ come in inverse pairs (see, for instance, Geoff Robinson's comment). But for $n\geq 3$, a complex orthogonal matrix need not be diagonalizable, and it seems nontrivial to determine what Jordan normal forms can occur depending on $n$. $\endgroup$ Jul 2, 2015 at 15:18
  • 1
    $\begingroup$ The paper "The Jordan canonical forms of complex and skew-symmetric matrices" by Horn and Merino classifies which Jordon blocks are similar to an orthogonal matrix with $\beta=I$. However I would like to identify $\beta$ as well in the process. $\endgroup$ Jul 3, 2015 at 5:24

Your Answer

By clicking “Post Your Answer”, you agree to our terms of service and acknowledge you have read our privacy policy.

Not the answer you're looking for? Browse other questions tagged or ask your own question.